Determinare p, q primi tali che p^3 - q^5 = (p + q)^2

Numeri interi, razionali, divisibilità, equazioni diofantee, ...
Rispondi
Avatar utente
HiTLeuLeR
Messaggi: 1874
Iscritto il: 01 gen 1970, 01:00
Località: Reggio di Calabria

Determinare p, q primi tali che p^3 - q^5 = (p + q)^2

Messaggio da HiTLeuLeR »

Problema: determinare ogni coppia $ (p, q) $ di primi naturali tali che $ p^3 - q^5 = (p+q)^2 $.
Avatar utente
peppeporc
Messaggi: 100
Iscritto il: 07 mar 2005, 17:20

Re: Determinare p, q primi tali che p^3 - q^5 = (p + q)^2

Messaggio da peppeporc »

HiTLeuLeR ha scritto:Problema: determinare ogni coppia $ (p, q) $ di primi naturali tali che $ p^3 - q^5 = (p+q)^2 $.
Visto che sbagliando s'impara provo a sbagliare questa...
tento per $ p=q $;
$ p^3-p^5=(p+p)^2 $
$ p^3-p^5-4p^2=0 $
divido per $ p^2 $
$ p-p^3-4=0 $
$ p-p^3=4 $, assurdo, quindi $ p\neq q $.
Tento per $ p=2 $:
$ 8-q^5=(2+q)^2 $;
$ 8-q^5=4+q^2+4q $;
$ q^5+q^2+4q=4 $;
$ q(q^4+q+4)=4 $, assurdo, assurdo poichè $ p $ può assumere solo il valore di $ 2 $ come divisore di $ 4 $, ma $ p\neq q $.
Tento per $ q=2 $:
$ p^3-32=(p+2)^2 $;
$ p^3-p^2-4p=36 $;
$ p(p^2-p-4)=36 $, assurdo poichè $ p $ può assumere solo il valore di $ 2 $ come divisore di $ 36 $, ma $ p\neq q $ e $ 3 $, ma andando a sosituire non ci troveremmo.
Ora è accertato che $ p $ e $ q $ sono entrambi dispari, pertanto $ p+q $ è pari e $ (p+q)^2\equiv 0\pmod{4} $. Aggiungendo $ 2q^5 $ ad ogni membro dell'equazione di partenza abbiamo
$ p^3+q^5=(p+q)^2+2q^5 $,
dove $ 2q^5\equiv 1\pmod{4} $
quindi $ (p+q)^2+2q^5\equiv 2\pmod{4} $.
Ora verifichiamo che
$ p^3\equiv q^5\equiv 3 oppure \equiv 2 \pmod{4} $
e le possibili configurazioni sono
$ p^3+q^5\equiv 2 oppure \equiv 0\pmod{4} $.

Se ho bestemmiato mi assumo le responsabilità di uno che si affaccia solo ora a questo mondo dopo tre anni di provinciali..mannaggia a me!! :P
(ho corretto su segnalazione fi HITLEuLeR)

Scatenatevi coi commenti, le correzioni e i rimproveri (ma non troppo pesanti) :P
Ultima modifica di peppeporc il 15 ago 2005, 17:03, modificato 1 volta in totale.
Tu chiamale, se vuoi, emozioni.
Avatar utente
HiTLeuLeR
Messaggi: 1874
Iscritto il: 01 gen 1970, 01:00
Località: Reggio di Calabria

Messaggio da HiTLeuLeR »

peppeporc ha scritto:[...] tento per $ p=q $: [...] $ p-p^3=4 $, assurdo, quindi $ p\neq q $. Tento per $ p=2 $: [...] $ q(q^4+q+4)=4 $, assurdo poichè $ p $ può assumere solo il valore di $ 2 $ come divisore di $ 4 $, ma $ p\neq q $.
Ok, fin qui è tutto corretto!
peppeporc ha scritto:Tento per $ q=2 $: [...] $ p(p^2-p-4)=36 $, assurdo poichè $ p $ può assumere solo il valore di $ 2 $ come divisore di $ 36 $, ma $ p\neq q $.
Ehm... Davvero $ 36 = 2^2 \cdot 3^2 $, quindi in linea di principio potrebbe anche essere $ p = 3 \neq q $. Senonché $ 3 \cdot (3^2 - 3 - 4) \neq 36 $, e questa possibilità va comunque esclusa.
peppeporc ha scritto:Ora è accertato che $ p $ e $ q $ sono entrambi dispari, pertanto $ p+q $ è pari e $ (p+q)^2\equiv 0\pmod{4} $.
Ok, anche questo è corretto!
peppeporc ha scritto:Aggiungendo $ 2q^5 $ ad ogni membro dell'equazione di partenza abbiamo $ p^3+q^5=(p+q)^2+2q^5 $, dove $ 2q^5\equiv 1\pmod{4} $
Eh, no! Questo è falso. Siccome infatti $ q \equiv 1 \bmod 2 $, allora $ q^4 \equiv 1 \bmod 4 $, e quindi $ 2q^5 \equiv 2q \equiv 2 \bmod 4 $. Il resto delle tue argomentazioni risulta di conseguenza compromesso. Mh, vedi un po' se riesci a trovare comunque il modo di concludere...
Avatar utente
peppeporc
Messaggi: 100
Iscritto il: 07 mar 2005, 17:20

Messaggio da peppeporc »

E' possibile continuando a seguire lo stesso discorso?
Tu chiamale, se vuoi, emozioni.
ma_go
Site Admin
Messaggi: 1906
Iscritto il: 01 gen 1970, 01:00

Messaggio da ma_go »

beh, le congruenze aiutano assai... non sono strettamente indispensabili, ma aiutano... io l'ho risolto usando anche quelle, casomai se nessuno si fa vivo con una soluzione... posso postarla, tra un paio di giorni...
Simo_the_wolf
Moderatore
Messaggi: 1053
Iscritto il: 01 gen 1970, 01:00
Località: Pescara

Messaggio da Simo_the_wolf »

io suggerirei di scrivere tutto mod p e mod q...
In questo modo, vedendo tutto mod q abbiamo che:
$ p^3 \equiv p^2 \pmod{q} $
cioe' $ p=kq+1 $
Guardando tutto mod p abbiamo invece che $ -q^5 \equiv q^2 \pmod{p} $ e quindi $ p|(q+1)(q^2-q+1) $.
Abbiamo 2 casi:
1) $ p|q+1 $ e quindi $ kq+1|q+1 $ e allora $ k=1 $ e allora $ p=q+1 $ e quindi $ p=3 $ e $ q=2 $ ma non funziona
2) $ p|q^2-q+1 $ e allora $ j(kq+1)=q^2-q+1 $ analizzando questa mod q abbiamo $ j\equiv1 \pmod{q} $ ma $ j \leq q $ altrimenti $ j(kq+1) $ sarebbe troppo grande. Ma allora $ j=1 $ e $ k=q-1 $ e allora $ p=q^2-q+1 $. Sostituendo all'inizio abbiamo:

$ (q^2-q+1)^3-q^5=(q^2+1)^2 $
Avatar utente
HiTLeuLeR
Messaggi: 1874
Iscritto il: 01 gen 1970, 01:00
Località: Reggio di Calabria

Messaggio da HiTLeuLeR »

Bene, Simo, questo è un modo. Certo l'equazione finale in $ q $ impone qualche calcolo, ma vabbè... Se oggi a dettare le regole del buon gusto in fatto di abbigliamento ci sono, fra gli altri, quei due simpatici di Dolce&Gabbana, allora non è certo mio diritto, e di nessun altro, star lì a blaterare invano, perso in vuoti discorsi filosofanti circa una presunta estetica canonizzata e il vago sentimento di un'eleganza che non s'indossa e di una bellezza che poco deve, o nulla, ai trucchi e al bisturi del chirurgo... :shock:
ma_go
Site Admin
Messaggi: 1906
Iscritto il: 01 gen 1970, 01:00

Messaggio da ma_go »

ok, ora posto il mio:
ragioniamo $ \mod 3 $, e distinguiamo due casi:

(i) $ p+q \equiv 0 $: si ha $ p^3 \equiv p $ e $ q^5 \equiv q $ (conseguenze del piccolo teorema di fermat) e $ (p+q)^2 \equiv 0 $, quindi $ p-q \equiv 0 $. quindi $ p \equiv q \equiv 0 $, quindi $ p = q = 3 $, che non mi pare essere soluzione...

(ii) $ p+q \equiv \pm 1 $: si ha $ p-q \equiv 1 $, quindi almeno uno dei due tra $ p $ e $ q $ è $ 3 $. però $ p > q $, quindi si hanno due ulteriori "casini":
(ii/a) $ p=3 $, $ q = 2 $, che non funge.
(ii/b) $ q = 3 $, da cui $ p^3 - p^2 - 6p - 252 $. fattorizzando $ 252 $, l'unica soluzione prima possibile è $ p=7 $, che guarda caso funge: $ 7^3 - 3^5 = (7+3)^2 $.

(notare, questo dà una soluzione a una delle diofantee di poliwhirl postate recentemente)
Avatar utente
HiTLeuLeR
Messaggi: 1874
Iscritto il: 01 gen 1970, 01:00
Località: Reggio di Calabria

Messaggio da HiTLeuLeR »

Davvero bella, ma_go, l'ho molto apprezzata. :o
Avatar utente
HiTLeuLeR
Messaggi: 1874
Iscritto il: 01 gen 1970, 01:00
Località: Reggio di Calabria

Messaggio da HiTLeuLeR »

:arrow: Il problema proviene dall'Olimpiade Nazionale Russa del 1997, è giusto dirlo!
Rispondi